www.vorhilfe.de
Vorhilfe

Kostenlose Kommunikationsplattform für gegenseitige Hilfestellungen.
Hallo Gast!einloggen | registrieren ]
Startseite · Forum · Wissen · Kurse · Mitglieder · Team · Impressum
Forenbaum
^ Forenbaum
Status Englisch
  Status Grammatik
  Status Lektüre
  Status Korrekturlesen
  Status Übersetzung
  Status Sonstiges (Englisch)

Gezeigt werden alle Foren bis zur Tiefe 2

Navigation
 Startseite...
 Neuerdings beta neu
 Forum...
 vorwissen...
 vorkurse...
 Werkzeuge...
 Nachhilfevermittlung beta...
 Online-Spiele beta
 Suchen
 Verein...
 Impressum
Das Projekt
Server und Internetanbindung werden durch Spenden finanziert.
Organisiert wird das Projekt von unserem Koordinatorenteam.
Hunderte Mitglieder helfen ehrenamtlich in unseren moderierten Foren.
Anbieter der Seite ist der gemeinnützige Verein "Vorhilfe.de e.V.".
Partnerseiten
Weitere Fächer:

Open Source FunktionenplotterFunkyPlot: Kostenloser und quelloffener Funktionenplotter für Linux und andere Betriebssysteme
Forum "Physik" - Lösung der Maxwellgleichungen
Lösung der Maxwellgleichungen < Physik < Naturwiss. < Vorhilfe
Ansicht: [ geschachtelt ] | ^ Forum "Physik"  | ^^ Alle Foren  | ^ Forenbaum  | Materialien

Lösung der Maxwellgleichungen: Frage (beantwortet)
Status: (Frage) beantwortet Status 
Datum: 17:04 Sa 04.06.2011
Autor: hydendyden

Hallo,
ich soll zeigen, dass die folgenden Felder die Maxwellgleichungen lösen:
[mm] \vec{E}(\vec{r},t)=\vec{E_0}sin(\vec{k}\cdot \vec{r}-\omega [/mm] t)
[mm] \vec{B}(\vec{r},t)=\frac{1}{ck}(\vec{k}\times \vec{E_0})sin(\vec{k}\cdot \vec{r}-\omega [/mm] t) mit [mm] \rho=0, \vec{j}=\vec{0}. [/mm]
Die ersten drei Gleichungen machen keine Probleme, die vierte will jedoch einfach nicht hinhauen:
rot [mm] \vec{B}-\frac{1}{c}\partial_t \vec{E}=\frac{1}{ck} \vec{k}\times(\vec{k} \times \vec{E_0})cos(\vec{k}\cdot \vec{r}-\omega t)+\frac{1}{c} \vec{E_0}\omega cos(\vec{k}\cdot \vec{r}-\omega [/mm] t)
[mm] =(\frac{1}{ck}(\vec{k}(\vec{k}\cdot \vec{E_0})-\vec{E_0}(\vec{k} \cdot \vec{k}))+\frac{\omega}{c}\vec{E_0})cos(\vec{k}\cdot \vec{r}-\omega [/mm] t)
[mm] =(-\frac{1}{ck}\vec{E_0}k^2+\frac{\omega}{c}\vec{E_0})cos(\vec{k}\cdot \vec{r}-\omega [/mm] t) (weil [mm] \vec{k} \cdot \vec{E_0}=0) [/mm]
[mm] =\frac{1}{c}(-k+\omega)\vec{E_0}cos(\vec{k}\cdot \vec{r}-\omega [/mm] t)
Und das ist natürlich alles andere als 0.
Es wär nett wenn mir jemand sagen könnte, wo mein Fehler liegt.

Gruß

Ich habe diese Frage in keinem Forum auf anderen Internetseiten gestellt.

        
Bezug
Lösung der Maxwellgleichungen: Antwort
Status: (Antwort) fertig Status 
Datum: 20:16 Sa 04.06.2011
Autor: Kroni

Hi,

in deiner Maxwell-Gleichung fehlt vor der zeitlichen Ableitung des E-Feldes noch ein $1/c$, da

[mm] $\nabla \times [/mm] B - [mm] \frac{1}{c^2}\partial_t [/mm] E =0$

Dann kannst du am Ende deinen Vorfaktor, der [mm] $\omega$ [/mm] und $k$ enthaelt mit Hilfe der Dispersionsrelation

[mm] $\omega=ck$ [/mm] zu Null diskutieren.

LG

Kroni



Bezug
                
Bezug
Lösung der Maxwellgleichungen: Frage (beantwortet)
Status: (Frage) beantwortet Status 
Datum: 21:31 Sa 04.06.2011
Autor: hydendyden

Hallo,
danke für deine Antwort. Woher nimmst du den Faktor [mm] \frac{1}{c}? [/mm] Ich finde die Maxwellgleichungen nur in der Form ohne diesen Faktor, z.B. []hier.

Gruß

Bezug
                        
Bezug
Lösung der Maxwellgleichungen: Antwort
Status: (Antwort) fertig Status 
Datum: 21:44 Sa 04.06.2011
Autor: chrisno

$E [mm] \ne [/mm] D$ und $B [mm] \ne [/mm] H$

Bezug
                                
Bezug
Lösung der Maxwellgleichungen: Frage (reagiert)
Status: (Frage) reagiert/warte auf Reaktion Status 
Datum: 21:50 Sa 04.06.2011
Autor: hydendyden

Hallo,
das ist mir schon klar, ich hatte nur auf die schnelle nichts anderes gefunden. Ich hab hier den Fließbach liegen und da steht auch drin:
[mm] rot\vec{B}(\vec{r},t)-\frac{1}{c}\frac{\partial \vec{E}(\vec{r},t)}{\partial t}=\frac{4\pi}{c}\vec{j}(\vec{r},t). [/mm]

Bezug
                                        
Bezug
Lösung der Maxwellgleichungen: Mitteilung
Status: (Mitteilung) Reaktion unnötig Status 
Datum: 22:21 Sa 04.06.2011
Autor: hydendyden

Ich glaube ich habe jetzt das Problem gefunden: Die Aufgabe ist für das elektromagnetische cgs-System gedacht. Wir benutzen aber das Gauß-System. Schreibt man im B-Feld anstatt [mm] \frac{1}{ck} [/mm] nur [mm] \frac{1}{k}, [/mm] passt alles. Eigentlich hätte mir schon bei der dritten Maxwellgleichung auffallen sollen, dass etwas nicht stimmt, nur hab ich hier zufälligerweise den Faktor [mm] \frac{1}{c} [/mm] vergessen, sodass alles passte. Vielen Dank nochmal für die Antworten.

Gruß

Bezug
Ansicht: [ geschachtelt ] | ^ Forum "Physik"  | ^^ Alle Foren  | ^ Forenbaum  | Materialien


^ Seitenanfang ^
www.englischraum.de
[ Startseite | Forum | Wissen | Kurse | Mitglieder | Team | Impressum ]